Use the diagram below to answer the questions. Line q contains points J, K, and M. Point P is above line q between points K and M. A line connects points M and P. Another line connects points P and K. Point L is above point J. A line starts at point K and extends through point L. Which are shown on the diagram? Check all that apply. Line segment J L Ray K M Line J K Ray P K AngleLJK Ray M J

Answers

Answer 1

Answer:

2nd 3rd last one

Step-by-step explanation:

Answer 2

Angles, lines, rays, and segments can be found in a plane. The terms shown on the diagram are: Ray KM, Line JK, and Ray MJ

What are lines, rays, and segments?

A line is an unending, straight path that has no endpoints and travels in both directions along a plane. A line segment is a finitely long portion of a line with two endpoints. A line segment that goes on forever in one direction is known as a ray.

To identify the terms shown on the diagram, we need to note the following: A ray has no endpoint; it is represented with an arrow on one side line segment has two endpoints.

It is represented with dots on both sides. An angle is a space between intersecting lines, line segments, or rays. There is no connection between J and L. So, JL is not a line segment

Ray KM

KM has a dot at point K and an arrow that points in the direction of M. So, KM is a ray.

Line JK

JK has dots on both ends (at J and K). So, JK is a line.

Ray PK

PK has dots on both sides (at P and K). So, PK is a line; not a ray.

Angle LJK

There is no connection between points L, J, and K. Hence, LJK is not an angle.

Ray MJ

MJ has a dot at point M and an arrow that points in the direction of J. So, MJ is a ray.

The diagram is attached with the answer below.

Read more about lines, points, and rays at:

brainly.com/question/13486760

#SPJ5

Use The Diagram Below To Answer The Questions. Line Q Contains Points J, K, And M. Point P Is Above Line

Related Questions

If f(x) = 4x + 15, then f(-3) = ?

Answers

Answer:

[tex]\Huge \boxed{3}[/tex]

Step-by-step explanation:

The function is given :

f(x) = 4x + 15

For f(-3), the input for the function f(x) is -3.

Replace the x variable with -3.

f(-3) = 4(-3) + 15

Evaluate.

f(-3) = -12 + 15

f(-3) = 3

The output for f(-3) is 3.

Answer: f(-3) = 3

Step-by-step explanation: Notice that f is a function of x.

So we want to find f(-3).

We find f(-3) by plugging -3 in for x,

everywhere that x appears in the function.

So we have 4(-3) + 15.

4(-3) is -12 so we have -12 + 15 which is 3.

So f(-3) is 3.

how do you find the length of the hypotenuse when you have only the length of the altitude of the hypotensuse and a length of a leg?

Answers

Answer:

By using The Pythagorean Theorem:

[tex]/Hypotenuse/^{2} = /Length of altitude/^{2} + /Length of leg/^{2} \[/tex]

/Hypotenuse/ = [tex]\sqrt\ /Length of altitude/^{2} + /Length of leg/^{2} \}[/tex]

Step-by-step explanation:

The Pythagorean theorem states that: Given a Right-angled triangle, the square of the hypotenuse equals the sum of squares of the other two sides ( Here, being the length of the altitude and length of leg). That is,

[tex]/Hypotenuse/^{2} = /Length of altitude/^{2} + /Length of leg/^{2} \[/tex] and hence,

/Hypotenuse/ = [tex]\sqrt\ /Length of altitude/^{2} + /Length of leg/^{2} \}[/tex]

For example, If the length of the altitude is 4m and the length of leg is 3m. Using The Pythagorean theorem, the length of the hypotenuse will be

[tex]/Hypotenuse/^{2} = /Length of altitude/^{2} + /Length of leg/^{2} \\\/Hypotenuse/ = \sqrt{/Length of altitude/^{2} + /Length of leg/^{2}} \\/Hypotenuse/ = \sqrt{4^{2} + 3^{2} }[/tex]

[tex]/Hypotenuse/ = \sqrt{16+9} \\/Hypotenuse/ = \sqrt{25} \\/Hypotenuse/ = 5m[/tex]

The length of the hypotenuse for the given example will be 5m.

This is how to find the length of an hypotenuse.

use the formula S = 40,000 (1.06)t to calculate your salary after 4 years. Round your answer to the nearest dollar.



a. $42,400


b. $44,944


c. $47,641


d. $50,499

Answers

Answer:

d. $50,499

Step-by-step explanation:

Given:

S = 40,000 (1.06)^t

Where,

t=4 years

S=40,000(1.06)^4

=40,000(1.26247696)

=50,499.0784

To the nearest dollar

S=$50,499

The answer is d. $50,499

what is mean absolute deviation (MAD) and how do I find it? ​

Answers

Answer with explanation:The mean absolute deviation (MAD) of a data set is the mean of the absolute distances from the mean.

Steps to find MAD:

Step 1. Calculate mean([tex]\overline{x}[/tex]) of the data using formula: [tex]\overline{x}=\dfrac{\sum x}{n}[/tex] ,  where x denotes data points and n is the number of data points.

Step 2. Calculate distance of each data point from mean :

Distance = [tex]|x-\overline{x}|[/tex]

Step 3. Divide  distance of each data point from mean by n:

MAD = [tex]\dfrac{\sum |x-\overline{x}|}{n}[/tex] , which is the final computation to find MAD.

PLEASE help me with this question!!! REALLY URGENT!

Answers

Answer:

The third table is the correct answer

Step-by-step explanation:

Here in this question, we are concerned with determine which of the tables correctly represents what an exponential function is.

An exponential function is a function of the form;

y = x^n

where the independent variable x in this case is raised to a certain exponent so as to give the results on the dependent variable axis (y-axis)

In the table, we can see that we have 2 segments, one that contains digits 1,2 and so on while the other contains purely the powers of 10.

Now, let’s set up an exponential outlook;

y = 10^x

So we have;

1 = 10^0

10 = 10^1

1/10 = 10^-1

1000 = 10^3

1/100 = 10^-2

We can clearly see here that we have an increase in the value of y, depending on the value of the exponent.

However it is only this table that responds to this successive correctness as the other tables in the answer do have a point where they fail.

For example;

10^-2 is not 10 which makes the fourth table wrong

10^4 is not 100 which makes the first table wrong

we have same error on second table too

PLEASE help me solve this question! No nonsense answers please!

Answers

Answer:

[tex]\boxed{\sf Option \ 1}[/tex]

Step-by-step explanation:

The profit is revenue (R ) - costs (C ).

Subtract the expression of costs (C ) from revenue (R ).

[tex]10x-0.01x^2-(2x+100)[/tex]

Distribute negative sign.

[tex]10x-0.01x^2-2x-100[/tex]

Combine like terms.

[tex]8x-0.01x^2-100[/tex]

The first option has a positive 100, which is wrong.

The rest options are right, when we expand brackets the result is same.

20 POINTS! ***CORRECT*** ANSWER GETS BRAINLIEST!!!!
The fraction model below shows the steps that a student performed to find a quotient.
Which statement best interprets the quotient?
A. There are 5 1/6 three-fourths in 4 1/8
B. There are 5 1/6 three and one-eights in 3/4
C. There are 5 1/2 three and one-eights in 3/4
D. There are 5 1/2 three-fourths in 4 1/8

Answers

Answer:

(D) There are [tex]5 \frac{1}{2}[/tex]  three-fourths in [tex]4 \frac{1}{8}[/tex]

Step-by-step explanation:

We can see that in this model, the student tried to put [tex]\frac{3}{4}[/tex] into  [tex]4 \frac{1}{8}[/tex]. We know this because the top of Step 2 is  [tex]4 \frac{1}{8}[/tex] and he is counting how many fourths in the bottom.

So this becomes the division statement:

[tex]4 \frac{1}{8} \div \frac{3}{4}[/tex].

We can convert  [tex]4 \frac{1}{8}[/tex] into a mixed number by multiplying 8 and 4, then adding 1.

[tex]\frac{33}{8} \div \frac{3}{4}[/tex].

Multiply by the reciprocal:

[tex]\frac{33}{8} \cdot \frac{4}{3} = \frac{132}{24}[/tex]

Which simplifies down to

[tex]\frac{11}{2}[/tex], which is just [tex]5 \frac{1}{2}[/tex] in improper form.

Hope this helped!

Answer:

D

Step-by-step explanation:

Helppppp!!!! Thank you

Answers

Greetings from Brasil...

In a triangle the sum of the internal angles is 180 °.... Thus,

Ô = 180 - 30

Ô = 60

The desired area is the area of the rectangle triangle, minus the area of the circular sector whose angle 60

A1 = area of the rectangle triangle

TG B = OA/AB

AB = OA / TG B

AB = 6 / TG 30

AB = 6√3

A1 = (AB . OA)/2

A1 = (6√3 . 6)/2

A1 = 18√3

A2 = area of the circular sector

(rule of 3)

 º                       area

360    ------------   πR²

60     ------------    X

X = 60πR²/360

X = 6π

So,

A2 = 6π

Then the area shaded is:

A = A1 - A2

A = 18√3 - 6π

How many triangles exist with the given side lengths? 2mm,6mm,10mm

Answers

Answer:

Zero

Step-by-step explanation:

2+6=8 which means it can't be. It has to be a length higher than 10

Kapil deposited Rs. 1600 in a bank on 1st January 2005. Find the amount in his bank account on 1st January 2006, if the bank pays interest at 8% per annum and the interest is calculated every year on 30th June and 31st December.

Answers

Answer:

SI=PRT/100

=10000*5*42/12*100

=1750

SI=1750

TOTAL AMOUNT=PRINCIPLE+SI

=10000+1750

=101750

In a naval engagement, one-third of the fleet was captured, one-sixth was sunk, and two ships were destroyed by fire. One-seventh of the surviving ships were lost in a storm after the battle. Finally, the twenty-four remaining ships sailed home. How many ships were in the fleet before the engagement?

Answers

Answer:

60 ships.

Step-by-step explanation:

Let the total number of ships in the naval fleet be represented by x

One-third of the fleet was captured = 1/3x

One-sixth was sunk = 1/6x

Two ships were destroyed by fire = 2

Let surviving ships be represented by y

One-seventh of the surviving ships were lost in a storm after the battle = 1/7y

Finally, the twenty-four remaining ships sailed home

The 24 remaining ships that sailed home =

y - 1/7y = 6/7y of the surviving fleet sailed home.

Hence

24 = 6/7y

24 = 6y/7

24 × 7/ 6

y = 168/6

y = 28

Therefore, total number of ships that survived is 28.

Surviving ships lost in the storm = 1/7y = 1/7 × 28 = 4

Total number of ships in the fleet(x) =

x = 1/3x + 1/6x + 2 + 28

Collect like terms

x - (1/3x + 1/6x) = 30

x - (1/2x) = 30

1/2x = 30

x = 30 ÷ 1/2

x = 30 × 2

x = 60

Therefore, ships that were in the fleet before the engagement were 60 ships.

The projected worth (in millions of dollars) of a large company is modeled by the equation w = 206(1.1) t. The variable t represents the number of years since 2000. What is the projected annual percent of growth, and what should the company be worth in 2011? A. 10%; $534.31 million B. 11%; $646.52 million C. 10%; $587.74 million D. 11%; $226.60 million

Answers

Answer:

Hey There!! The Correct answer is: The equation is w = 241(1.06)t

And here variable t represents the number of years since 2000.

In 2001 means t=2001 -2000 = 1

So we plug 1 for t in the given expression , that is w = 241(1.06)1 = 241 * 1.06 = 255.46

Therefore in 2001, it should be worth to 255.46.

And in the given expression 1.06=1 +0.06, where 0.06 is the annual percent of growth that is 6 % .

Hope It Helped!~ ♡

ItsNobody~ ☆

The projected annual percent of growth is 10% and the company worth in 2011 will be $587.74 millions. Then the correct option is C.

What is an exponent?

Consider the function:

y = a (1 ± r) ˣ

Where x is the number of times this growth/decay occurs, a = initial amount, and r = fraction by which this growth/decay occurs.

If there is a plus sign, then there is exponential growth happening by r fraction or 100r %.

If there is a minus sign, then there is exponential decay happening by r fraction or 100r %.

The projected worth (in millions of dollars) of a large company is modeled by the equation is given as,

[tex]\rm w = 206\times (1.10)^t\\\\w = 206\times (1+0.10)^t[/tex]

Then the projected annual percent of growth is 10%.

The variable t represents the number of years since 2000.

Then the company worth in 2011 will be

w = 206 × 1.1¹¹

w = $587.74 millions

The projected annual percent of growth is 10% and the company worth in 2011 will be $587.74 millions.

Then the correct option is C.

More about the exponent link is given below.

https://brainly.com/question/5497425

#SPJ2

Write the equation of a circle with a center at (12, 6) and a radius of 6.

Answers

Answer:

(x-12)² + (y-6)² = 36 (Option C)

Step-by-step explanation:

use circle formula

(x-h)² + (y-k)²= r²

h= 12 and k= 6 and r= 6

(x-12)² + (y-6)² = 6²

6 squared = 36 (6·6)

(x-12)² + (y-6)² = 36

What is the value of w? inscribed angles (Image down below)

Answers

Answer:

w = 100°

Step-by-step explanation:

Opposite angles in an inscribed quadrilateral in a circle are supplementary.

Therefore, [tex] w + 80 = 180 [/tex]

Subtract 80 from both sides

[tex] w + 80 - 80 = 180 - 80 [/tex]

[tex] w = 100 [/tex]

The value of w = 100°

A students wants to report on the number of movies her friends watch each week. The collected date are below:

0, 0, 1, 1, 2, 2, 2, 14

which measure of center is most appropriate for this situation and what's its value?

A.) Median; 1.5
B.) Median; 3
C.) Mean; 1.5
D.) Mean; 3​

Answers

Answer:

A.) median; 1.5

Step-by-step explanation:

Hello!

The median is the number that is in the middle when the numbers are listed from least to greatest

0, 0, 1, 1, 2, 2, 2, 14

We can take one from both sides till there are one or two numbers left

0, 1, 1, 2, 2, 2

1, 1, 2, 2

1, 2

If there are two numbers left we add them then divide by 2 to get the median

1 + 2 = 3

3 / 2 = 1.5

The answer is A.) median; 1.5

Hope this helps!

If the sphere shown above has a radius of 17 units, then what is the approximate volume of the sphere?
A.
385.33 cubic units
B.
4,913 cubic units
C.
6,550.67 cubic units
D.
3,275.34 cubic units

Answers

Answer:

20582.195 units

Step-by-step explanation:

This problem is on the mensuration of solids.

A sphere is a solid shape.

Given data

radius of sphere = 17 units

The volume of a sphere can be expressed as below

[tex]volume = \frac{4}{3}\pi r^3[/tex]

Substituting our data into the expression we have

[tex]volume = \frac{4}{3}*3.142*17^3[/tex]

[tex]volume = \frac{4}{3}*3.142*4913\\\\volume = \frac{61746.584}{3}= 20582.195[/tex]

The volume of the sphere is given as

20582.195 units

Please Help me with this math question

Answers

The answer is 26 while the exponent is 8.

The quotient of x^2+x-6/x^2-6x+5*x^2+2x-3/x^2-7x+10 has ___ in the numerator and ______ in the denominator.

Answers

Answer:

So the quotient of [tex]\frac{x^{2} + x - 6}{x^{2} -6x + 5} X \frac{x^{2} + 2x - 3}{x^{2} -7x + 10}[/tex] has (x + 3)² in the numerator and (x + 5)² in the denominator.

Step-by-step explanation:

[tex]\frac{x^{2} + x - 6}{x^{2} -6x + 5} X \frac{x^{2} + 2x - 3}{x^{2} -7x + 10}[/tex]

Factorizing the expressions we have

[tex]\frac{x^{2} + 3x -2x - 6}{x^{2} -x - 5x + 5} X \frac{x^{2} + 3x - x - 3}{x^{2} -2x -5x + 10}[/tex]

[tex]\frac{x(x + 3)- 2(x + 3)}{x(x -1) - 5(x - 1)} X \frac{x(x + 3) - 1(x + 3)}{x(x - 2) - 5(x - 2)}[/tex]

[tex]\frac{(x + 3)(x - 2)}{(x - 5)(x - 1)}X\frac{(x + 3)(x - 1)}{(x - 2)(x - 5)}[/tex]

Cancelling out the like factors, (x -1) and (x - 2), we have

[tex]\frac{(x + 3)(x + 3)}{(x - 5)(x - 5)}[/tex]

= [tex]\frac{(x + 3)^{2} }{(x + 5)^{2} }[/tex]

So the quotient of [tex]\frac{x^{2} + x - 6}{x^{2} -6x + 5} X \frac{x^{2} + 2x - 3}{x^{2} -7x + 10}[/tex] has (x + 3)² in the numerator and (x + 5)² in the denominator.

Dr. Potter provides vaccinations against polio and measles. Each polio vaccination consists of 6 doses, and each measles vaccination consists of 3 doses. Last year, Dr. Potter gave a total of 60 vaccinations that consisted of a total of 225 doses. How many more measles vaccines did Mr. Potter give than polio? Show All Work !!

Answers

Answer:

The number of measles vaccines that Dr. Potter give than polio vaccines is 30

Step-by-step explanation:

The parameters given are;

The number of doses given in a polio vaccine = 6 doses

The number of doses given in a measles vaccine = 3 doses

The number of vaccinations given by Dr. Potter last year = 60 vaccinations

The number of doses given in the 60 vaccinations = 225 doses

Let the number of polio vaccine given last year by Dr. Potter = x

Let the number of measles vaccine given last year by Dr. Potter = y

Therefore, we have;

6 × x + 3 × y = 225.......................(1)

x + y = 60.......................................(2)

From equation (2), we have;

x = 60 - y

Substituting the derived value for x in equation (1), we get;

6 × x + 3 × y = 225

6 × (60 - y) + 3 × y = 225

360 - 6·y + 3·y = 225

360 - 225 = 6·y - 3·y

135 = 3·y

y = 45

x = 60 - y = 60 - 45 = 15

Therefore;

The number of polio vaccine given last year by Dr. Potter = 15

The number of measles vaccine given last year by Dr. Potter = 45

The number of measles vaccines that Dr. Potter give than polio vaccines = 45 - 15 = 30 vaccines.

The number of measles vaccines that Dr. Potter give than polio vaccines =  30 vaccines.

Use distributive property to evaluate the expression 5(4/1/5)

Answers

Answer:

21

Step-by-step explanation:

4[tex]\frac{1}{5}[/tex] = [tex]\frac{21}{5}[/tex]

5 × [tex]\frac{21}{5}[/tex] = (5×21)/5

[tex]\frac{105}{5}[/tex] = 21

Please answer this question now

Answers

Answer:

156.6 square yards

Step-by-step explanation:

To find the surface area of the pyramid, find the area of each surface and add them together.

formula for area of a triangle = 1/2(b·h)

1. There are three triangles with a base of 9 and a height of 9

1/2(9·9) = 40.5

Multiply by the three triangles

40.5 · 3 = 121.5

2. There is one triangle with a base of 9 and a height of 7.8

1/2(9·7.8) = 35.1

3. Add the areas of all surfaces

121.5 + 35.1 = 156.6

What is the main difference between simplifying and solving? Which one gives you a value for a variable? How do you know the difference?

Answers

Answer:

when you simplify you continue until you get to the simplest form but when you solve you continue until you get an answer. Solving gives you a value for a variable. You mean simplify and get 2x - 10 but when you solve you continue until you get x as 5

Step-by-step explanation:

Answer: ok, so simplifying is when you make something less complex or complicated. Solving means an expression can be used for representating  the solutions. for Example, say if you have the equation x+y=2x-1 is solved for the unknown x by the expression x=y+1. solving gives you the value for the variable. you know the difference by when you are simplifying you are trying to make the problem less complicated or less complex. and when you are solving you are trying to find the answer to the problem..

Step-by-step explanation:

Tatenda takes ttt seconds to mow a square meter of lawn and Ciara takes ccc seconds to mow a square meter of lawn. Tatenda mows 700700700 square meters of lawn per week and Ciara mows 750750750 square meters of lawn per week. Which expressions can we use to describe how many more seconds Tatenda spends than Ciara spends mowing lawns during 444 weeks? Choose 2 answers: Choose 2 answers: (Choice A) A 4(750c-700t)4(750c−700t)4, left parenthesis, 750, c, minus, 700, t, right parenthesis (Choice B) B 3000c+2800t3000c+2800t3000, c, plus, 2800, t (Choice C) C 2800t-3000c2800t−3000c2800, t, minus, 3000, c (Choice D) D 4(700t-750c)4(700t−750c)4, left parenthesis, 700, t, minus, 750, c, right parenthesis (Choice E) E 4(700t+750c)4(700t+750c)

Answers

Answer:

C.) 4(750c-700t) ; D.) 2800t - 3000c

Step-by-step explanation:

Time taken :

Tatenda = t sec/m^2

Ciara = c sec/m^2

Tatenda = 700m^2 per week

Ciara = 750m^2 per week

Which expressions can we use to describe how many more seconds Tatenda spends than Ciara spends mowing lawns during 4

Total Time taken over four weeks :

Tatenda = 4(t * 700) = 4(700c)

Ciara = 4(c * 750) = 4(750c )

Number of seconds Tatenda spends more than Ciara : meaning Tatenda spends more seconds than

Tatenda - Ciara

4(700t) - 4(750c) = 4(700t - 750c)

4(700t - 750c)

Or

4(700t - 750c) = 2800t - 3000c

2800t - 3000c


Let f (x) = |2). Write a function g whose graph is a vertical shrink by a factor of
followed by a translation 2 units up of the graph of f.

Answers

Answer:

This is poorly written, so i will answer it as it was:

"Let f (x) = |2). Write a function g(x) whose graph is a vertical shrink by a factor of  A, followed by a translation 2 units up of the graph of f."

I don't really know what you do mean by I2), so i will answer it in a general way.

First, we do a vertical shrink of factor A.

A must be a number smaller than one and larger than zero., then if g(x) is a vertical shrink of factor A of the graph of f(x), we have that:

g(x) = A*f(x)

As 0 < A < 1

We will have that the graph of g(x) is a vertical compression of the graph of f(x)

Now we do a vertical shift of 2 units up.

A general vertical shift of N units up is written as:

g(x) = f(x) + N

Where N is a positive number.

So in our case, we have:

g(x) = A*f(x) + 2.

Where you will need to replace the values of A and f(x) depending on what the actual question says,

How many solutions does the nonlinear system of equations graphed below
have?
y
10+
-10
10
-10
A. One
B. Two
0
O
C. Four
O
D. Zero

Answers

Answer:

D. zero

Step-by-step explanation:

Since the graphs do not intersect, there are zero solutions.

The number of solutions on the graph is zero

How to determine the number of solutions?

The graph shows a linear equation (the straight line) and a non linear equation (the curve)

From the graph, we can see that the straight line and the curve do not intersect

This means that the graph do not have any solution

Hence, the number of solutions on the graph is zero

Read more about non-linear graphs at:

https://brainly.com/question/16274644

#SPJ5

PLEASE HELP Polynomial Graph Studies Polynomials are great functions to use for modeling real-world scenarios where different intervals of increase and decrease happen. But polynomial equations and graphs can be trickier to work with than other function types. In mathematical modeling, we often create an equation to summarize data and make predictions for information not shown on the original display. In this activity, you’ll create an equation to fit this graph of a polynomial function. Part A Describe the type of function shown in the graph. Part B What are the standard form and the factored form of the function? Part C What are the zeros of the function? Part D Use the zeros to find all of the linear factors of the polynomial function. Part E Write the equation of the graphed function f(x), where a is the leading coefficient. Use the factors found in part D. Express the function as the product of its leading coefficient and the expanded form of the equation in standard form. Part F Use the y-intercept of the graph and your equation from part E to calculate the value of a. Part G Given what you found in all of the previous parts, write the equation for the function shown in the graph.

Answers

Answer:

Here's what I get  

Step-by-step explanation:

Part A

The graph shows a polynomial of odd degree. It is probably a third-degree polynomial — a cubic equation.

Part B

The standard form of a cubic equation is

y = ax³ + bx² + cx + d

The factored form of a cubic equation is

y = a(x - b₁)(x² + b₂x + b₃)

If you can factor the quadratic, the factored form becomes

y = a(x - c₁)(x - c₂)(x - c₃)

Part C

The zeros of the function are at x = -25, x = - 15, and x = 15.

Part D

The linear factors of the function are x + 25, x + 15, and x - 15.

Part E

y = a(x + 25)(x + 15)(x - 15) = a(x + 25)(x² - 225)  

y = a(x³ + 25x² - 225x - 5625)

Part F

When x = 0, y = 1.

1 = a[0³ +25(0)² - 225(0) - 5625] = a(0 + 0 - 0 -5625) = -5625a

a = -1/5625

Part G

[tex]y = -\dfrac{1}{5625}( x^{3} + 25x^{2} - 225x - 5625)\\\\y = \mathbf{ -\dfrac{1}{5625} x^{3} - \dfrac{1}{225}x^{2} + \dfrac{1}{25} x + 1}[/tex]

Answer

Actually, the answer should be -0.0007(x+20)(x+5)(x-15)

Step-by-step explanation:

This is continuing off of the previous answer

PART C

The zeros should be (15,0), (-5,0), and (-20,0)

PART D

x - 15, x + 5, and x + 20

PART E

a(x - 15)(x + 5)(x + 20)

Standard: [tex]a(x^{3} + 10x^{2} -275x-1500)[/tex]

PART F

The y-intercept is at (0,1), so we replace the x's with 0:

1 =[tex][(0)x^{3} +10(0)x^{2} -275(0)-1500][/tex] and this gives us (0+0-0-1500) which also equals -1500

Then we do [tex]\frac{1}{-1500}[/tex] which gives us -0.0006 repeating which rounds to -0.0007

a= -0.0007

PART G

Just place the numbers where they should go and your answer is

y =-0.0007(x + 20)(x + 5)(x - 15)  

the placement for (x + 20) (x + 5) and (x - 15) doesn't matter as long as they are behind -0.0007

Need help on the third question. how do i generalise the number of ways to win.(check the attatchment)

Answers

Answer:

  2n+2 ways to win

Step-by-step explanation:

You generalize by observing patterns in the way you solve the smaller problems.

The number of winning moves is 2n+2: the total of the number of diagonals, columns, and rows.

For an n×n board, there are 2 full-length diagonals, n columns, and n rows, hence 2+n+n = 2n+2 ways to win.

Could someone clarrify this for me Factor completely 3x^2 + 2x − 1. (3x + 1)(x − 1) (3x + 1)(x + 1) (3x − 1)(x + 1) (3x − 1)(x − 1)

Answers

Answer:

(3x-1) (x+1)

Step-by-step explanation:

3x^2 + 2x − 1

3x^2 factors into 3x and x

-1 factors into -1 and 1

We want a postive 2x

(3x-1) (x+1)

Answer:

(3x-1)(x+1)

Step-by-step explanation:

3x² + 2x − 1

when factorizing , first look at the constant number( in this case it is 1 prime number), then find the GCF if found.

(3x        )(x      )     first step : 3x*x=3x^2

(3x-      ) (x+   )      the sign for the constant is minus so the factoring has to be minus and plus on each side

(3x-1)(x+1)   look at the 2x it has positive sign, means the sign is plus:

3x-1

x+1

                        regular standard multiplication

3x(x)-1(x)+1(3x)-1

3x²+2x-1

What is the 8th term for this sequence -3, -12, -48, -192,...

Answers

Answer:

-49152

Step-by-step explanation:

The pattern is:

multiply for 4

then:

-3, -12, -48, -192, -768, -3072, -12288, -49152

(a²b²-c²)(a²b²+c²)
simplify​

Answers

Answer:

a⁴b⁴ - c⁴

Step-by-step explanation:

The difference of squares formula states that (a - b)(a + b) = a² - b². In this case, a = a²b² and b = c² so a² - b² = (a²b²)² - (c²)² = a⁴b⁴ - c⁴.

Answer:

a^4b^4 - c^4.

Step-by-step explanation:

(a²b²-c²)(a²b²+c²)

Difference of 2 squares:

= (a²b²)^2 - (c²)^2

= a^4b^4 - c^4.

Other Questions
A cross-sectional view of a log shows a concentric pattern. Which part of the stem gives rise to this pattern? The business case for why companies should act in a socially responsible manner includes: Select one: a. It generates internal benefits including employee recruiting, workforce retention, training, and improved worker productivity b. It reduces the risk of reputation-damaging incidents c. It is in the best interest of shareholders and offers potential for increased buyer patronage d. All of the above Which Supreme Court case resulted in due process rights for adults being available for juveniles accused of crimes?Hazelwood v. KuhlmeierTinker v. Des MoinesIn re GaultBrown v. Board of Education The Hirt & Block mutual fund has assets of $147 million, liabilities of $7 million and 7 million shares outstanding. The shares trade at $21.60 per share. What is the percentage load fee? Correct Inc. is a publicly traded firm with 100 million diluted shares outstanding trading at $37.50 per share. The company has $1 billion of debt outstanding with a cost of debt at 6.5% at a marginal tax rate of 40%. The company has $100 million of cash on its balance sheet. What is the enterprise value of Correct Inc. A circle has a circumference of 153.86153.86153, point, 86 units.What is the radius of the circle? What are examples of career fields Skills USA prepares students for? Check all that apply.health scienceO educationagricultureconstructionmanufacturingtransportationinformation technologypublic safety Kate's Diner offers one breakfast item, a breakfast special. The market price for this meal is $5. At her profit-maximizing level of output, Kate's average variable cost is $4 per meal. Her average total cost is $6 per meal. Since the market is perfectly competitive, Kate should ________. Kevin has a BMI of 28.5. How many calories should he eliminate from his diet every day to achieve weight loss? a.600 calories b.400 calories c.100 calories d.200 calories e.300 calories "An individual who is 25 years from retirement has $500,000 to invest today. He is risk tolerant and is looking to withdraw $80,000 per year once he retires. Which asset allocation is BEST for this client?" . One sample has M = 18 and a second sample has M = 14. If the pooled variance for the two samples is 16, what is the value of Cohens d? A company has a selling price of $2,150 each for its printers. Each printer has a 2 year warranty that covers replacement of defective parts. It is estimated that 3% of all printers sold will be returned under the warranty at an average cost of $157 each. During November, the company sold 37,000 printers, and 470 printers were serviced under the warranty at a total cost of $62,000. The balance in the Estimated Warranty Liability account at November 1 was $32,500. What is the company's warranty expense for the month of November You plan to take your hair blower to Europe, where the electrical outlets put out 240 V instead of the 120 V seen in the United States. The blower puts out 1700 W at 120 V.Required:a. What could you do to operate your blower via the 240V line in Europe? which one is it?b. What current will your blower draw from a European outlet?c. What resistance will your blower appear to have when operated at 240 ? Gig Harbor Boating is the wholesale distributor of a small recreational catamaran sailboat. Management has prepared the following summary data to use in its annual budgeting process:Budgeted unit sales 500Selling price per unit $1,970Cost per unit $1,460Variable selling and administrative expenses (per unit) $ 50Fixed selling and administrative expenses (per year) $196,000Interest expense for the year $ 13,000Required:Prepare the companys budgeted income statement using an absorption income statement format shown below.Down Under Products, Ltd., of Australia has budgeted sales of its popular boomerang for the next four months as follows:Sales in UnitsApril 74,000 May 85,000 June 114,000 July 92,000 The company is now in the process of preparing a production budget for the second quarter. Past experience has shown that end-of-month inventory levels must equal 10% of the following months sales. The inventory at the end of March was 7,400 units.Required:Prepare a production budget for the second quarter; in your budget, show the number of units to be produced each month and for the quarter in total.Garden Depot is a retailer that is preparing its budget for the upcoming fiscal year. Management has prepared the following summary of its budgeted cash flows:1st Quarter 2nd Quarter 3rd Quarter 4th QuarterTotal cash receipts $310,000 $430,000 $360,000 $380,000Total cash disbursements$365,000 $335,000 $325,000 $345,000The companys beginning cash balance for the upcoming fiscal year will be $25,000. The company requires a minimum cash balance of $10,000 and may borrow any amount needed from a local bank at a quarterly interest rate of 3%. The company may borrow any amount at the beginning of any quarter and may repay its loans, or any part of its loans, at the end of any quarter. Interest payments are due on any principal at the time it is repaid. For simplicity, assume that interest is not compounded.Required:Complete the company's cash budget for the upcoming fiscal year. (Cash deficiency, repayments, and interest, should be indicated by a minus sign.)The management of Mecca Copy, a photocopying center located on University Avenue, has compiled the following data to use in preparing its budgeted balance sheet for next year:Ending BalancesCash ?Accounts receivable $ 8,500Supplies inventory $ 4,700Equipment $ 36,000Accumulated depreciation $ 14,600Accounts payable $ 2,200Common stock $ 5,000Retained earnings ?The beginning balance of retained earnings was $32,000, net income is budgeted to be $16,300, and dividends are budgeted to be $2,700.Required:Prepare the companys budgeted balance sheet. (Amounts to be deducted should be indicated by a minus sign.) PLS HELP:Find all the missing elements: Somebody please help its urgent!!!!In the tug of war game, none of the teams won. What can you conclude about the forces of the two teams ? Write all the evidence to support your answer. Which phrase best completes the sentence? Margarita: Fulmos ___unas faldas, unos cinturones de marca, in vistido pata y un lente nuevo para mi camara. Joaquin used two types of flour in a muffin recipe. How much flour did he use in all? Solve any way you choose. How could you incorporate listening for needs, purpose, or concern to create value in the conversation (including describing what that would look like)? The intermolecular forces present in CH 3NH 2 include which of the following? I. dipole-dipole II. ion-dipole III. dispersion IV. hydrogen bonding